Jump to content

Nguyenhuyen_AG's Content

There have been 785 items by Nguyenhuyen_AG (Search limited from 05-06-2020)



Sort by                Order  

#636122 Inequalities From 2016 Mathematical Olympiads

Posted by Nguyenhuyen_AG on 27-05-2016 - 23:34 in Bất đẳng thức - Cực trị

Bài 37 (Turkmenistan RMO). Nếu $a,b,c$ là độ dài ba cạnh của tam giác thì

\[\sqrt{\frac{a}{b+c-a}}+\sqrt{\frac{b}{c+a-b}}+\sqrt{\frac{c}{a+b-c}} \geqslant  3.\]
 




#641892 Inequalities From 2016 Mathematical Olympiads

Posted by Nguyenhuyen_AG on 23-06-2016 - 15:48 in Bất đẳng thức - Cực trị

Bài 44 (BMOJ TST). Cho ba số thực $a \geqslant b \geqslant 1 \geqslant c\geqslant 0$ thỏa mãn điều kiện $a+b+c=3.$

  1. Chứng minh rằng $2 \leqslant ab + bc +ca \leqslant 3.$
     
  2. Chứng minh rằng \[\frac{24}{a^3+b^3+c^3} + \frac{25}{ab+bc+ca} \geqslant 14.\] Đẳng thức xảy ra khi nào ?



#648325 Inequalities From 2016 Mathematical Olympiads

Posted by Nguyenhuyen_AG on 07-08-2016 - 00:11 in Bất đẳng thức - Cực trị

Bài 54 (Azerbaijan JBMO TST). Với $a,b,c$ là ba số thực dương thỏa mãn điều kiện $a+b+c=3.$ Chứng minh rằng
\[2(ab+ac+bc)-3abc \geqslant a \sqrt{\frac{b^2+c^2}{2}}+b \sqrt{\frac{c^2+a^2}{2}}+c \sqrt{\frac{a^2+b^2}{2}}.\]




#624579 Inequalities From 2016 Mathematical Olympiads

Posted by Nguyenhuyen_AG on 03-04-2016 - 20:03 in Bất đẳng thức - Cực trị

Bài 17 (Azerbaijan Junior Mathematical Olympiad). Với $x,\,y,\,z$ là ba số thực khác $0.$ Chứng minh rằng $$\sqrt {x^2+\frac {1}{y^2}}+ \sqrt {y^2+\frac {1}{z^2}}+ \sqrt {z^2+\frac {1}{x^2}}\geq 3\sqrt {2}. $$

Bài 18 (Macedonia National Olympiad). Cho $n \;(n \geqslant 3)$ số thực dương $a_1,\,a_2,\,\ldots,a_n$ thỏa mãn điều kiện

$$\frac{1}{1+a_1^4} + \frac{1}{1+a_2^4} + \cdots + \frac{1}{1+a_n^4} = 1.$$

Chứng minh rằng $$a_1a_2 \cdots a_n \ge (n-1)^{\frac n4}.$$

Bài 19 (Cyprus TST). Giả sử $a,\,b,\,c$ là độ dài ba cạnh của tam giác $ABC$ và thỏa mãn điều kiện
$$a\sqrt{8}+b\sqrt{6}+c\sqrt{2}\ge 4\sqrt{a^2+b^2+c^2},$$
Chứng minh rằng $ABC$ là tam giác vuông.

 




#626228 Inequalities From 2016 Mathematical Olympiads

Posted by Nguyenhuyen_AG on 10-04-2016 - 00:35 in Bất đẳng thức - Cực trị

Bài 18 (Macedonia National Olympiad). Cho $n \;(n \geqslant 3)$ số thực dương $a_1,\,a_2,\,\ldots,a_n$ thỏa mãn điều kiện

$$\frac{1}{1+a_1^4} + \frac{1}{1+a_2^4} + \cdots + \frac{1}{1+a_n^4} = 1.$$

Chứng minh rằng $$a_1a_2 \cdots a_n \ge (n-1)^{\frac n4}.$$
 




#678809 Topic BẤT ĐẲNG THỨC ôn thi vào lớp 10 THPT 2017 - 2018

Posted by Nguyenhuyen_AG on 27-04-2017 - 23:01 in Bất đẳng thức và cực trị

Làm trội lên :

(Lê Khánh Sỹ, tạp chí toán học rumania)

Cho $a, b, c \geq 0$; $c=min\left \{ a,b,c \right \}$ . Chứng minh rằng:

$$\frac{a^{2}+b^{2}+c^{2}}{ab+bc+ca}+ \frac{8abc}{(a+b)(b+c)(c+a)} \geq 2 + \frac{2c(a-b)^{2}}{3(a+b)(b+c)(c+a)}$$

 

Ta có

\[\text{VT-VP} = \frac{\left[(ab+2bc+5ca)(a-c)+(2a^2+3ab+3bc)(b-c)\right](a-b)^2+3c^2(a+b)(a-c)(b-c)}{3(a+b)(b+c)(c+a)(ab+bc+ca)} \geqslant 0.\]




#679199 Topic BẤT ĐẲNG THỨC ôn thi vào lớp 10 THPT 2017 - 2018

Posted by Nguyenhuyen_AG on 02-05-2017 - 02:02 in Bất đẳng thức và cực trị

Bài 33: (Khánh Sỹ)

Cho các số thực không âm $a,b,c$ thỏa mãn $a+b+c=3$. Chứng minh rằng:

$$ \sum_{cyc} \frac{a^{2}}{a+3} + \frac{ab+bc+ca}{4} \geq \frac{3}{2} $$

 

Ta có

\[\sum \frac{a^{2}}{a+3} + \frac{ab+bc+ca}{4} - \frac{3}{2} = \frac{1}{12(a+3)(b+3)(c+3)} \sum a(3a^2+bc)(b-c)^2 \geqslant 0.\]

 

$\boxed{\textbf{ BÀI TOÁN 31 }}$ 

Cho $ a,b,c\ge 0$ . Chứng Minh Rằng:

$3(a^2+b^2+c^2)(a^2b^2+b^2c^2+c^2a^2)\geq(a^2+ab+b^2)(b^2+bc+c^2)(c^2+ca+a^2)$

 

\[\text{Vế trái  - Vế phải} = \frac14\sum \left[(a+b-c)^2+5a^2+5b^2+c^2\right]c^2(a-b)^2 \geqslant 0.\]




#677981 Topic BẤT ĐẲNG THỨC ôn thi vào lớp 10 THPT 2017 - 2018

Posted by Nguyenhuyen_AG on 19-04-2017 - 00:27 in Bất đẳng thức và cực trị

$\boxed{8}$ [Trần Quốc Anh] Cho $a,b,c$ là các số thực không âm thỏa mãn $a+b+c=2$. Chứng minh rằng:

\[(a^2+ab+b^2)(b^2+bc+c^2)(c^2+ca+a^2) \le 3 \]

 

Bất đẳng thức cần chứng minh tương đương với

\[\frac{3}{64}(a+b+c)^6 \geqslant (a^2+ab+b^2)(b^2+bc+c^2)(c^2+ca+a^2),\]

hoặc

\[\frac{5}{16}(a+b+c)^2 \sum ab(a-b)^2+\frac{1}{64}\left(\sum a^2-2\sum bc\right)^2\left(3\sum a^2 + 10 \sum bc \right) + \frac{1}{16}abc(a+b+c)^3 \geqslant 0.\]

Từ đó suy điều phải chứng minh.

 

P/s. Bài này không phải của Trần Quốc Anh đâu.




#679401 Topic BẤT ĐẲNG THỨC ôn thi vào lớp 10 THPT 2017 - 2018

Posted by Nguyenhuyen_AG on 03-05-2017 - 21:49 in Bất đẳng thức và cực trị

tại sao anh tìm đc đánh giá trên vậy?

 

Anh dùng hệ số bất định kết hợp với đạo hàm.

 

Có cách giải nào ngoài phương pháp Đ giá đại diện sau khi dùng đạo hàm như anh k ạ?

Cách này em biết rồi nhưng ở đây ta cần một lời giải phù hợp THCS hơn ạ. 

Ps: chỉ là góp ý nho nhỏ,

 

Đổi biến $(a,b,c) \to \left(\frac{bc}{a^2},\frac{ca}{b^2},\frac{ab}{c^2}\right)$ và đánh giá bằng bất đẳng thức Holder

\[\left(\sum \frac{a^2}{\sqrt{a^4+6a^2bc+2b^2c^2}}\right)^2 \sum a^2(a^4+6a^2bc+2b^2c^2) \geqslant (a^2+b^2+c^2)^3.\]




#679393 Topic BẤT ĐẲNG THỨC ôn thi vào lớp 10 THPT 2017 - 2018

Posted by Nguyenhuyen_AG on 03-05-2017 - 21:28 in Bất đẳng thức và cực trị

BÀI TOÁN 41( Sưu tầm):  Cho $a,b,c\ge0$ thỏa mãn $abc=1$

CMR: $\frac{1}{\sqrt{2a^2+6a+1}}+\frac{1}{\sqrt{2b^2+6b+1}}+\frac{1}{\sqrt{2c^2+6c+1}}\geq1$

 

Chỉ cần chứng minh

\[\frac{1}{\sqrt{2a^2+6a+1}} \geqslant \frac{1}{a^{10/9} + a^{5/9} + 1}.\]




#679632 Topic BẤT ĐẲNG THỨC ôn thi vào lớp 10 THPT 2017 - 2018

Posted by Nguyenhuyen_AG on 05-05-2017 - 22:42 in Bất đẳng thức và cực trị

Aảo quá .Có ai còn cách nào tự nhiên hơn không?

 

Cách dùng bất đẳng thức cổ điển thì có nhưng gõ lâu hơn phân tích bình phương.

 

Viết bất đẳng thức cần chứng minh lại như sau

\[(a^2+b^2+c^2)^3 \geqslant 9abc(a^3+b^3+c^3).\]

Giả sửa $a = \max\{a,b,c\}.$ Áp dụng bất đẳng thức AM-GM, ta có

\[9abc(a^3+b^3+c^3) \leqslant \left(ab+ca+\frac{a^3+b^3+c^3}{3a}\right)^3.\]

Ta cần chứng minh

\[a^2+b^2+c^2 \geqslant ab+ca+\frac{a^3+b^3+c^3}{3a},\]

thu gọn thành

\[\frac{(2a-b-c)(a^2+b^2+c^2-ab-bc-ca)}{3a} \geqslant 0.\]

Hiển nhiên đúng nên ta có điều phải chứng minh.

 




#679589 Topic BẤT ĐẲNG THỨC ôn thi vào lớp 10 THPT 2017 - 2018

Posted by Nguyenhuyen_AG on 05-05-2017 - 16:34 in Bất đẳng thức và cực trị

Bài  46 (sưu tầm) Cho các số thực a,b,c >0 và abc=1.

      Chứng minh rằng :  (a2+b2+c2)≥ 9(a3+b3+c3)

 

Ta có

\[(a^2+b^2+c^2)^3 - 9(a^3+b^3+c^3) = \frac13\sum(ab+2bc+2ca+c^2)(a-b)^4+\frac12\sum\left[(a^2+b^2)^2+5c^4\right](a-b)^2.\]




#680649 Topic BẤT ĐẲNG THỨC ôn thi vào lớp 10 THPT 2017 - 2018

Posted by Nguyenhuyen_AG on 14-05-2017 - 14:20 in Bất đẳng thức và cực trị

Bài toán 61(sưu tầm)

Cho a,b,c là các số thực không âm, đôi một khác nhau. CMR:

$(ab+bc+ca)\left [ \frac{1}{(a-b)^{2}}+\frac{1}{(b-c)^{2}}+\frac{1}{(c-a)^{2}} \right ]\geqslant 4$

 

Ta có

\[\text{Vế trái  -Vế phải} = \sum \frac{ab\left[c(a+b-c)+a^2-3ab+b^2\right]^2}{(a-b)^2(b-c)^2(c-a)^2} \geqslant 0.\]

P/s. Anh nhớ Doflamingo bị Hải quân bắt rồi sao còn ở đây post bài nhỉ?




#569777 THƯ MỜI HỌP MẶT 5/7/2015 tại Tp. Hồ Chí Minh

Posted by Nguyenhuyen_AG on 04-07-2015 - 03:38 in Góc giao lưu

Anh có số của Võ Anh Đức, HCV IMO 2013 có được không. :D



#569776 THƯ MỜI HỌP MẶT 5/7/2015 tại Tp. Hồ Chí Minh

Posted by Nguyenhuyen_AG on 04-07-2015 - 03:36 in Góc giao lưu

Sao trúng ngày xem chung kết Copa vậy, 3h xem tới ~ 5h, xong ngủ đến chiều luôn. Quốc Anh qua rước anh nhé.



#651905 Vincal 570ES PLUS II giải sai nghiệm ?

Posted by Nguyenhuyen_AG on 29-08-2016 - 22:21 in Giải toán bằng máy tính bỏ túi

May quá Maple vẫn đúng :)

 

20169736ed47-7777-45ca-bee3-bc9379904808




#614662 Bổ đề hoán vị

Posted by Nguyenhuyen_AG on 13-02-2016 - 09:21 in Tài liệu, chuyên đề, phương pháp về Bất đẳng thức

Chào các thành viên của VMF, nhân dịp Tết Bính Thân 2016 mình xin tặng cho các thành viên trên VMF một chuyên đề bất đẳng thức của mình. Chuyên đề này ... à mà thôi mình dở văn nên xin bỏ qua đoạn này, mọi người đọc và cho tác giả ý kiến. :)

 

BỔ ĐỀ HOÁN VỊ

 

1. Mở đầu

 

Năm 2008 trên diễn đàn toán học Art of Problem Solving anh Võ Quốc Bá Cẩn đề xuất một bổ đề khá thú vị sau:

Bổ đề. Với mọi số thực dương $a,\,b,\,c$ thỏa mãn $a+b+c=1,$ đặt $q = ab+bc+ca\,(1 \geqslant 3q).$ Chứng minh rằng
\begin{equation} \label{(bodehoanvi)}
\frac{a}{b}+\frac{b}{c}+\frac{c}{a} \geqslant \frac{2(27q^2-9q+1)}{9q^2-2q+(1-3q)\sqrt{q(1-3q)}}+\frac{1}{q}-6.
\end{equation}

 

(Võ Quốc Bá Cẩn)

 

Lời giải. Đặt $p=a+b+c,\,q=ab+bc+ca$ và $r=abc$ (ta sẽ thống nhất cách đặt này cho cả bài viết) khi đó
\[(a-b)^2(b-c)^2(c-a)^2 = p^2q^2 - 4q^3 + 2p(9q - 2p^2)r - 27r^2.\]
Ta có đánh giá
\[\begin{aligned} 2(ab^2+bc^2+ca^2) & = \sum ab(a+b) + (a-b)(b-c)(c-a) \\ &= pq - 3r + (a-b)(b-c)(c-a) \\& \geqslant pq - 3r - \sqrt{(a-b)^2(b-c)^2(c-a)^2} \\& = pq - 3r - \sqrt{p^2q^2 - 4q^3 + 2p(9q - 2p^2)r - 27r^2},\end{aligned}\]
vì thế
\[ab^2+bc^2+ca^2 \geqslant \frac{pq - 3r - \sqrt{p^2q^2 - 4q^3 + 2p(9q - 2p^2)r - 27r^2}}{2}.\]
Mặt khác từ giả thiết ta được $p = 1$ cho nên
\[\frac{a}{b}+\frac{b}{c}+\frac{c}{a} = \frac{ab^2+bc^2+ca^2}{abc} \geqslant \frac{q-3r-\sqrt{q^2-4q^3+2(9q-2)r-27r^2}}{2r}.\]
Xét hàm số
\[f\,(r) = \frac{q-3r-\sqrt{q^2-4q^3+2(9q-2)r-27r^2}}{2r},\]
tính đạo hàm
\[f^{'}(r) = \frac{q^2-4q^3+(9q-2)r-q\sqrt{q^2-4q^3+2(9q-2)r-27r^2}}{2r^2\sqrt{q^2-4q^3+2(9q-2)r-27r^2}},\]
do đó phương trình $f^{'}(r) = 0$ có nghiệm
\[r = r_0 = \frac{q^2\left[9q^2-2q+(1-3q)\sqrt{q(1-3q)}\right]}{27q^2-9q+1}.\]
Lập bảng biến thiên ta được $f(r) \geqslant f(r_0).$ Biến đổi
\[\begin{aligned} f(r_0) & = \frac{q - 3r_0 - \sqrt{q - 4q^3 + 2(9q - 2)r_0 - 27r^2_0}}{2r_0} \\& = \frac{q-3r_0 - \dfrac{q^2 - 4q^3 + (9q - 2)r_0}{q}}{r_0} \\& = \frac{2q^3 + (1 - 6q)r_0}{qr_0} = \frac{2q^2}{r_0} + \frac{1}{q} - 6 \\& = \frac{2(27q^2 - 9q + 1)}{9q^2 - 2q + (1 - 3q)\sqrt{q(1 - 3q)}} + \frac{1}{q} - 6.\end{aligned}\]
Đây chính là điều phải chứng minh.

Nhận xét.

 

  1. Ở thời điểm đó bổ đề trên là một bài toán rất khó, do hình thức khá cồng kềnh, không đẹp mắt (có căn thức nằm ở mẫu số) nên không nhận được sự quan tâm của nhiều người, sau này vào năm 2011 anh Lê Hữu Điền Khuê (Nesbit) mới đưa ra một chứng minh khác trên Diễn Đàn Toán Học.
  2. Đây là một kết quả rất chặt với vô số các trường hợp để đẳng thức xảy ra và cũng là dạng chặt nhất trong lớp các bài toán có dạng \[f\left(\frac{a}{b},\,\frac{b}{c},\,\frac{c}{a},\,a+b+c,\,ab+bc+ca\right) \geqslant 0.\]
  3. Bổ đề này sẽ giúp chúng ta sẽ giải quyết được rất nhiều bài toán khó sau đây (đã từng là unsolve suốt một thời gian dài trên AoPS).

2. Các bài toán áp dụng

Bài toán 1. Cho $a,\,b,\,c$ là ba số thực dương. Chứng minh rằng
\[\frac{a}{b}+\frac{b}{c}+\frac{c}{a} + \frac{28(ab+bc+ca)}{(a+b+c)^2} \geqslant 12.\]

 

(Võ Quốc Bá Cẩn)

 

Lời giải. Do tính thuần nhất của bài toán nên ta có thể chuẩn hóa $p=1,$ khi đó
\[\frac{28(ab+bc+ca)}{(a+b+c)^2} = \frac{28q}{p^2} = 28q.\]
Áp dụng bổ đề \eqref{(bodehoanvi)} ta đưa bài toán về chứng minh
\[\frac{2(27q^2-9q+1)}{9q^2-2q+(1-3q)\sqrt{q(1-3q)}} + \frac{1}{q} - 6 + 28q \geqslant 12,\]
hay là
\begin{equation} \label{lab1}
\frac{2(27q^2-9q+1)}{9q^2-2q+(1-3q)\sqrt{q(1-3q)}} + \frac{1}{q} + 28q \geqslant 18.
\end{equation}
Bất đẳng thức lúc này chỉ còn một biến nên hai công cụ đầu tiên mà chúng ta nghĩ đến là khảo sát hàm hoặc quy đồng phân tích nhân tử. Tuy nhiên biểu thức này khi lấy đạo hàm sẽ cho ra một kết quả “rất khủng” còn nếu phân tích nhân tử thì thì biểu thức thu được cũng không khá hơn mấy vì sự xuất hiện của căn thức ở mẫu.

Viết \eqref{lab1} lại như sau
\begin{equation} \label{lab2}
\frac{2(27q^2 - 9q + 1)}{9q^2 - 2q + (1 - 3q) \cdot q \cdot \sqrt{\frac{1 - 3q}{q}}} + \frac{1}{q} + 28q \geqslant 18.
\end{equation}
Đặt $x = \sqrt{\frac{1-3q}{q}} \geqslant 0$ thì $q = \frac{1}{x^2+3},$ bất đẳng thức \eqref{lab2} trở thành
\[\frac{2(x^2+3x+3)}{x+1} + x^2 + \frac{28}{x^2+3} \geqslant 15.\]
Xét hiệu hai vế ta được
\[\frac{2(x^2+3x+3)}{x+1} + x^2 + \frac{28}{x^2+3} - 15 = \frac{(x^3+5x^2+3x+1)(x-1)^2}{(x+1)(x^2+3)} \geqslant 0.\]
Đẳng thức xảy ra khi và chỉ khi và chỉ khi
\[\frac{a}{\sqrt{7}-\tan\frac{\pi}{7}} = \frac{b}{\sqrt{7}-\tan\frac{2\pi}{7}} = \frac{c}{\sqrt{7}-\tan\frac{4\pi}{7}}.\]
Vậy ta có điều phải chứng minh.

Bài toán 2. Cho $a,\,b,\,c$ là ba số thực dương. Chứng minh rằng
\[\frac{a}{b}+\frac{b}{c}+\frac{c}{a} + \frac{7(ab+bc+ca)}{a^2+b^2+c^2} \geqslant \frac{17}{2}.\]

 

(Võ Quốc Bá Cẩn)

 

Lời giải. Tương tự như trên ta cũng chuẩn hóa $p=1,$ khi đó
\[\frac{7(ab+bc+ca)}{a^2+b^2+c^2} = \frac{7q}{p^2-2q} = \frac{7q}{1-2q}.\]
Áp dụng bổ đề \eqref{(bodehoanvi)} ta đưa bài toán về chứng minh
\[\frac{2(27q^2-9q+1)}{9q^2-2q+(1-3q)\sqrt{q(1-3q)}} + \frac{1}{q} - 6 + \frac{7q}{1-2q} \geqslant \frac{17}{2},\]
hay là
\begin{equation} \label{lab3}
\frac{2(27q^2 - 9q + 1)}{9q^2 - 2q + (1 - 3q) \cdot q \cdot \sqrt{\frac{1 - 3q}{q}}} + \frac{1}{q} + \frac{7q}{1-2q} \geqslant \frac{29}{2}.
\end{equation}
Đặt $x = \sqrt{\frac{1-3q}{q}} \geqslant 0$ thì $q = \frac{1}{x^2+3},$ bất đẳng thức \eqref{lab3} trở thành
\[\frac{2(x^2+3x+3)}{x+1}+x^2+\frac{7}{x^2+1} \geqslant \frac{23}{2},\]
tương đương với
\[\frac{(2x^3+10x^2+9x+3)(x-1)^2}{2(x+1)(x^2+1)} \geqslant 0.\]
Đẳng thức xảy ra khi và chỉ khi
\[\frac{a}{\sqrt{7}-\tan\frac{\pi}{7}} = \frac{b}{\sqrt{7}-\tan\frac{2\pi}{7}} = \frac{c}{\sqrt{7}-\tan\frac{4\pi}{7}}.\]
Bài toán được chứng minh.

 

Nhận xét. Bài toán này là hệ quả của bài toán 1. Thật vậy vì
\[\frac{a}{b}+\frac{b}{c}+\frac{c}{a} \geqslant 12 - \frac{28(ab+bc+ca)}{(a+b+c)^2},\]
nên ta chỉ cần chứng minh
\[12 - \frac{28(ab+bc+ca)}{(a+b+c)^2} + \frac{7(ab+bc+ca)}{a^2+b^2+c^2} \geqslant \frac{17}{2},\]
hay là
\[\frac{7(a+b+c)^2}{2(a^2+b^2+c^2)} \geqslant \frac{28(ab+bc+ca)}{(a+b+c)^2},\]
hoặc
\[8(ab+bc+ca)(a^2+b^2+c^2) \leqslant (a+b+c)^4.\]
Áp dụng bất đẳng thức AM-GM, ta có
\[\begin{aligned}8(ab+bc+ca)(a^2+b^2+c^2) \leqslant  \left[2(ab+bc+ca) + (a^2+b^2+c^2)\right]^2  = (a+b+c)^4.\end{aligned}\]
Khi đẳng thức xảy ra thì ta được đẳng thức khá đẹp mắt $a^2+b^2+c^2 = 2(ab+bc+ca).$

Bài toán 3. Cho $a,\,b,\,c$ là ba số thực dương. Chứng minh rằng
\[\left(\frac{a}{b}+\frac{b}{c}+\frac{c}{a}\right)^2 + \frac{70(ab+bc+ca)}{a^2+b^2+c^2} \geqslant 60.\]

 

(Nguyễn Văn Huyện)

 

Lời giải. Chuẩn hóa $p=1$ và áp dụng bổ đề \eqref{(bodehoanvi)} ta đưa bài toán về chứng minh
\begin{equation} \label{lab3.1}
\left[\frac{2(27q^2-9q+1)}{9q^2-2q+(1-3q)\sqrt{q(1-3q)}} + \frac{1}{q} - 6\right]^2 + \frac{70q}{1-2q} \geqslant 60.
\end{equation}
Đặt $x = \sqrt{\frac{1-3q}{q}} \geqslant 0$ thì $q = \frac{1}{x^2+3},$ bất đẳng thức \eqref{lab3.1} trở thành
\[\left(\frac{x^3+3x^2+3x+3}{x+1}\right)^2 + \frac{70}{x^2+1} \geqslant 60,\]
tương đương với
\[\frac{(x^6+8x^5+31x^4+84x^3+119x^2+76x+19)(x-1)^2}{(x^2+1)(x+1)^2} \geqslant 0.\]
Đẳng thức xảy ra khi và chỉ khi
\[\frac{a}{\sqrt{7}-\tan\frac{\pi}{7}} = \frac{b}{\sqrt{7}-\tan\frac{2\pi}{7}} = \frac{c}{\sqrt{7}-\tan\frac{4\pi}{7}}.\]
Bài toán được chứng minh.

Bài toán 4. Cho $a,\,b,\,c$ là ba số thực dương. Chứng minh rằng
\[\left(\frac{a}{b}+\frac{b}{c}+\frac{c}{a}\right)^2 + \frac{280(ab+bc+ca)}{(a+b+c)^2} \geqslant 95.\]

 

(Tạ Hồng Quảng)

 

Lời giải. Chuẩn hóa $p=1$ và áp dụng bổ đề \eqref{(bodehoanvi)} ta đưa bài toán về chứng minh
\begin{equation} \label{lab3.2}
\left[\frac{2(27q^2-9q+1)}{9q^2-2q+(1-3q)\sqrt{q(1-3q)}} + \frac{1}{q} - 6\right]^2 + 280q \geqslant 95.
\end{equation}
Đặt $x = \sqrt{\frac{1-3q}{q}} \geqslant 0$ thì $q = \frac{1}{x^2+3},$ bất đẳng thức \eqref{lab3.2} trở thành
\[\left(\frac{x^3+3x^2+3x+3}{x+1}\right)^2 + \frac{280}{x^2+3} \geqslant 95,\]
tương đương với
\[\frac{(x^6+8x^5+33x^4+100x^3+144x^2+88x+22)(x-1)^2}{(x^2+3)(x+1)^2} \geqslant 0.\]
Đẳng thức xảy ra khi và chỉ khi
\[\frac{a}{\sqrt{7}-\tan\frac{\pi}{7}} = \frac{b}{\sqrt{7}-\tan\frac{2\pi}{7}} = \frac{c}{\sqrt{7}-\tan\frac{4\pi}{7}}.\]
Bài toán được chứng minh.

Bài toán 5. Cho $a,\,b,\,c$ là ba số thực dương. Chứng minh rằng
\[\frac{a}{b}+\frac{b}{c}+\frac{c}{a} \geqslant \frac{a^2+b^2+c^2}{ab+bc+ca} + \frac{6(a^2+b^2+c^2)}{(a+b+c)^2}.\]

 

(Nguyễn Văn Quý)

 

Lời giải. Chuẩn hóa $p=1$ và áp dụng bổ đề \eqref{(bodehoanvi)} ta đưa bài toán về chứng minh
\begin{equation} \label{lab8}
\frac{2(27q^2-9q+1)}{9q^2-2q+(1-3q)\sqrt{q(1-3q)}} + \frac{1}{q} - 6 \geqslant \frac{1-2q}{q} + 6(1-2q).
\end{equation}
Đặt $x = \sqrt{\frac{1-3q}{q}} \geqslant 0$ thì $q = \frac{1}{x^2+3},$ bất đẳng thức \eqref{lab8} trở thành
\[\frac{x^3+3x^2+3x+3}{x+1} \geqslant \frac{(x^2+9)(x^2+1)}{x^2+3},\]
hay là
\[\frac{2x^2(x-1)^2}{(x+1)(x^2+3)} \geqslant 0.\]
Đẳng thức xảy ra khi và chỉ khi $a=b=c$ hoặc
\[\frac{a}{\sqrt{7}-\tan\frac{\pi}{7}} = \frac{b}{\sqrt{7}-\tan\frac{2\pi}{7}} = \frac{c}{\sqrt{7}-\tan\frac{4\pi}{7}}.\]
Chứng minh hoàn tất.

Bài toán 6. Cho $a,\,b,\,c$ là ba số thực dương. Chứng minh rằng
\[\frac{a}{b}+\frac{b}{c}+\frac{c}{a}+\frac{k(ab+bc+ca)}{a^2+b^2+c^2} \geqslant 3+k,\]
trong đó $k = 3\sqrt[3]{4} - 2.$

 

(Ji Chen)

Lời giải. Chuẩn hóa $p=1$ và áp dụng bổ đề \eqref{(bodehoanvi)} ta đưa bài toán về chứng minh
\begin{equation} \label{lab4}
\frac{2(27q^2-9q+1)}{9q^2-2q+(1-3q)\sqrt{q(1-3q)}} + \frac{1}{q} + \frac{kq}{1-2q} \geqslant 9 + k.
\end{equation}
Đặt $x = \sqrt{\frac{1-3q}{q}} \geqslant 0$ thì $q = \frac{1}{x^2+3}$ bất đẳng thức \eqref{lab4} trở thành
\[\frac{2(x^2+3x+3)}{x+1}+x^2+\frac{k}{x^2+1} \geqslant k+6,\]
hay là
\[\frac{x^2[x^3+3x^2+(1-k)x-k+3]}{(x+1)(x^2+1)} \geqslant 0.\]
Bất đẳng thức này đúng vì với $k = 3\sqrt[3]{4} - 2,$ thì
\[x^3+3x^2+(1-k)x-k+3 = \big(x + 1 + 2\sqrt[3]{2}\big)\big(x + 1 - \sqrt[3]{2} \big)^2 \geqslant 0.\]
Đẳng thức xảy ra khi và chỉ khi $x=0$ hoặc $x = \sqrt[3]{2} - 1,$ cụ thể

  • Nếu $x = 0$ thì $q = \frac{1}{3}$ kết hợp với $p = 1$ ta được $a = b = c.$
  • Nếu  $x = \sqrt[3]{2} - 1$ thì $q = \frac{2+\sqrt[3]{2}}{10}$ dẫn đến $r = \frac{3-\sqrt[3]{2}}{50},$ kết hợp với $p=1$ ta thấy $a,\,b,\,c$ lần lượt là ba nghiệm của phương trình \[t^3 - t^2 + \frac{2+\sqrt[3]{2}}{10}t - \frac{3-\sqrt[3]{2}}{50} = 0.\] Bằng Maple hoặc Wolframalpha ta tìm được \[\left\{ \begin{aligned} & a = \frac{1}{3}+\frac{1}{3}\sqrt{\frac{8-6\sqrt[3]{2}}{5}} \cos \left(\frac{1}{3} \arccos \sqrt{\frac{101-54\sqrt[3]{4}}{20}}\right) \\ & b =  \frac{1}{3}-\frac{1}{3}\sqrt{\frac{8-6\sqrt[3]{2}}{5}} \sin \left(\frac{\pi}{6}-\frac{1}{3} \arccos \sqrt{\frac{101-54\sqrt[3]{4}}{20}}\right)\\ & c = \frac{1}{3}-\frac{1}{3}\sqrt{\frac{8-6\sqrt[3]{2}}{5}} \sin \left(\frac{\pi}{6}+\frac{1}{3} \arccos \sqrt{\frac{101-54\sqrt[3]{4}}{20}}\right)\end{aligned}\right.\] cùng các hoán vị.

Bài toán được chứng minh.

Bài toán 7. Cho $a,\,b,\,c$ là ba số thực dương. Chứng minh rằng
\[\frac{a}{b}+\frac{b}{c}+\frac{c}{a}+k\geqslant \frac{(9+3k)(a^2+b^2+c^2)}{(a+b+c)^2},\]
với $k = 3\sqrt[3]{2} - 3.$

(Võ Quốc Bá Cẩn, Bách Ngọc Thành Công)

Lời giải. Chuẩn hóa $p=1$ và áp dụng bổ đề \eqref{(bodehoanvi)} ta đưa bài toán về chứng minh
\begin{equation} \label{lab5}
\frac{2(27q^2-9q+1)}{9q^2-2q+(1-3q)\sqrt{q(1-3q)}} + \frac{1}{q} - 6 + k \geqslant (9+3k)(1-2q).
\end{equation}
Đặt $x = \sqrt{\frac{1-3q}{q}} \geqslant 0$ thì $q = \frac{1}{x^2+3},$ bất đẳng thức \eqref{lab5} trở thành
\[\frac{x^3+3x^2+3x+3}{x+1} + k \geqslant \frac{(9+3k)(x^2+1)}{x^2+3}.\]
Với $k = 3\sqrt[3]{2} - 3,$ ta có
\[\frac{x^3+3x^2+3x+3}{x+1} + k-\frac{(9+3k)(x^2+1)}{x^2+3} = \frac{x^2\big(x+1+2\sqrt[3]{4}\big)\big(x+1-\sqrt[3]{4}\big)^2}{(x+1)(x^2+3)} \geqslant 0\]
Đẳng thức xảy ra khi và chỉ khi $a=b=c$ hoặc $a,\,b,\,c$ lần lượt là ba nghiệm của phương trình
\[t^3 - t^2 + \frac{2+\sqrt[3]{4}}{12} t - \frac{1}{36} = 0,\]
cụ thể
\begin{equation} \label{nghiembac3}
\left\{ \begin{aligned} & a = \frac{1}{3}+\frac{1}{3}\sqrt{2-\sqrt[3]{4}} \cos \left(\frac{1}{3} \arccos \sqrt{\frac{7-\sqrt[3]{2}}{4}}\right) \\ & b = \frac{1}{3} - \frac{1}{3}\sqrt{2-\sqrt[3]{4}} \sin \left(\frac{\pi}{6}-\frac{1}{3} \arccos \sqrt{\frac{7-\sqrt[3]{2}}{4}}\right) \\ & c = \frac{1}{3} - \frac{1}{3}\sqrt{2-\sqrt[3]{4}} \sin \left(\frac{\pi}{6}+\frac{1}{3} \arccos \sqrt{\frac{7-\sqrt[3]{2}}{4}}\right)\end{aligned}\right.
\end{equation}
cùng các hoán vị. Bài toán được chứng minh.

Bài toán 8. Cho $a,\,b,\,c$ là các số thực dương. Chứng minh rằng
\[\frac{a}{b}+\frac{b}{c}+\frac{c}{a} \geqslant \sqrt{9-k+\frac{k(a^2+b^2+c^2)}{ab+bc+ca}},\]
trong đó $k = 3\big(1+\sqrt[3]2\big)^2.$

(Võ Quốc Bá Cẩn)

Lời giải. Chuẩn hóa $p=1$ và áp dụng bổ đề \eqref{(bodehoanvi)} ta đưa bài toán về chứng minh
\begin{equation} \label{lab6}
\frac{2(27q^2-9q+1)}{9q^2-2q+(1-3q)\sqrt{q(1-3q)}} + \frac{1}{q} - 6 \geqslant \sqrt{9-k+\frac{k(1-2q)}{q}}.
\end{equation}
Đặt $x = \sqrt{\frac{1-3q}{q}} \geqslant 0$ thì $q = \frac{1}{x^2+3},$ bất đẳng thức \eqref{lab6} trở thành
\[\frac{2(x^2+3x+3)}{x+1}+x^2-3 \geqslant \sqrt{9 + kx^2},\]
hay là
\[\frac{x^3+3x^2+3x+3}{x+1} \geqslant \sqrt{9 + kx^2},\]
hoặc
\[(x^3+3x^2+3x+3)^2 \geqslant (9 + kx^2)(x+1)^2,\]
\[x^2[x^4+6x^3+(15-k)x^2+2(12-k)x-k+18] \geqslant 0.\]
Đặt
\[P = x^4+6x^3+(15-k)x^2+2(12-k)x-k+18,\]
ta sẽ chứng minh $P \geqslant 0.$ Thật vậy, với $k = 3\big(1+\sqrt[3]2\big)^2$ thì
\[P = \big(x+1-\sqrt[3]{2}+\sqrt[3]{4}\big)\big(x+3+\sqrt[3]{2}+\sqrt[3]{4}\big)\big(x+1-\sqrt[3]{4}\big)^2 \geqslant 0.\]
Đẳng thức xảy ra khi và chỉ khi $a=b=c$ hoặc $a,\,b,\,c$ lần lượt là ba nghiệm của phương trình
\[t^3 - t^2 + \frac{2+\sqrt[3]{4}}{12} t - \frac{1}{36} = 0.\]
Giải phương trình này ta được nghiệm \eqref{nghiembac3}. Chứng minh hoàn tất.

Bài toán 9. Cho $a,\,b,\,c$ là ba số thực dương. Chứng minh rằng
\[\frac{a}{b}+\frac{b}{c}+\frac{c}{a}+\frac{(4\sqrt{2}-4)(ab+bc+ca)}{a^2+b^2+c^2} \geqslant 4\sqrt{2}-2+\frac{a^2+b^2+c^2}{ab+bc+ca}.\]

(Nguyễn Văn Huyện)

Lời giải. Chuẩn hóa $p=1$ và áp dụng bổ đề \eqref{(bodehoanvi)} ta đưa bài toán về chứng minh
\[\frac{2(27q^2-9q+1)}{9q^2-2q+(1-3q)\sqrt{q(1-3q)}} + \frac{1}{q} - 6+\frac{(4\sqrt{2}-4)q}{1-2q} \geqslant 4\sqrt{2}-2+\frac{1-2q}{q},\]
hay là
\begin{equation} \label{lab7}
\frac{2(27q^2-9q+1)}{9q^2-2q+(1-3q)\sqrt{q(1-3q)}} +\frac{(4\sqrt{2}-4)q}{1-2q} \geqslant 4\sqrt{2}+2.
\end{equation}
Đặt $x = \sqrt{\frac{1-3q}{q}} \geqslant 0$ thì $q = \frac{1}{x^2+3},$ khi đó bất đẳng thức \eqref{lab7} trở thành
\[\frac{x^3+3x^2+3x+3}{x+1} \geqslant \frac{x^4+4\sqrt{2}x^2+3}{x^2+1}.\]
Xét hiệu hai vế ta được
\[\frac{x^3+3x^2+3x+3}{x+1} - \frac{x^4+4\sqrt{2}x^2+3}{x^2+1} = \frac{2x^2\big(x+1-\sqrt{2}\big)^2}{(x+1)(x^2+1)} \geqslant 0.\]
Đẳng thức xảy ra khi và chỉ khi $a=b=c$ hoặc $a,\,b,\,c$ lần lượt là ba nghiệm của phương trình
\[t^3-t^2+\frac{3+\sqrt{2}}{14}t - \frac{2+3\sqrt{2}}{196} = 0,\]
cụ thể
\[\left\{ \begin{aligned} & a = \frac{1}{3}+\frac{1}{3}\sqrt{\frac{10-6\sqrt{2}}{7}} \cos \left(\frac{1}{3} \arccos \sqrt{\frac{115-27\sqrt{2}}{196}}\right) \\ & b = \frac{1}{3}-\frac{1}{3}\sqrt{\frac{10-6\sqrt{2}}{7}} \sin \left(\frac{\pi}{6}-\frac{1}{3} \arccos \sqrt{\frac{115-27\sqrt{2}}{196}}\right) \\ & c = \frac{1}{3}-\frac{1}{3}\sqrt{\frac{10-6\sqrt{2}}{7}} \sin \left(\frac{\pi}{6}+\frac{1}{3} \arccos \sqrt{\frac{115-27\sqrt{2}}{196}}\right)\end{aligned}\right.\]
Bài toán được chứng minh.

Bài toán 10. Cho ba số thực dương $a,\,b,\,c$ thỏa mãn điều kiện
\[\frac{a}{b} + \frac{b}{c} + \frac{c}{a} = \frac{5}{2}\cdot\frac{a^2+b^2+c^2}{ab+bc+ca}.\]
Chứng minh rằng \[a^2+b^2+c^2 \geqslant 2(ab+bc+ca).\]

 

(Nguyễn Văn Huyện)

Lời giải. Chuẩn hóa $p=1$ từ giả thiết áp dụng bổ đề \eqref{(bodehoanvi)} ta có
\begin{equation} \label{lab10}
\frac{2(27q^2-9q+1)}{9q^2-2q+(1-3q)\sqrt{q(1-3q)}}+\frac{1}{q} - 6 \leqslant \frac{5(1-2q)}{2q}.
\end{equation}
Đặt $x = \sqrt{\frac{1-3q}{q}} \geqslant 0$ thì $q = \frac{1}{x^2+3},$ khi đó \eqref{lab10} trở thành
\[\frac{x^3+3x^2+3x+3}{x+1} \leqslant \frac{5(x^2+1)}{2},\]
tương đương với
\[\frac{(x - 1)(3x^2 + 2x + 1)}{x+1} \geqslant 0.\]
Suy ra $x \geqslant 1,$ hay là
\[(a+b+c)^2 \geqslant 4(ab+bc+ca),\]
hoặc
\[a^2+b^2+c^2 \geqslant 2(ab+bc+ca).\]
Đẳng thức xảy ra khi và chỉ khi $a=b=c$ hoặc
\[\frac{a}{\sqrt{7}-\tan\frac{\pi}{7}} = \frac{b}{\sqrt{7}-\tan\frac{2\pi}{7}} = \frac{c}{\sqrt{7}-\tan\frac{4\pi}{7}}.\]
Chứng minh hoàn tất.

Bài toán 11. Với $k \geqslant 0$ là một số thực cho trước và $a,\,b,\,c$ là ba số thực dương sao cho
\[\frac{a}{b}+\frac{b}{c}+\frac{c}{a} = (k + 1)^2 + \frac{2}{k+1}.\]
Chứng minh rằng
\[a^2 + b^2 + c^2 \leqslant (k^2+1)(ab+bc+ca).\]

 

(Nguyễn Văn Huyện, VMEO IV)

Lời giải. Chuẩn hóa $p=1$ từ giả thiết áp dụng bổ đề \eqref{(bodehoanvi)} ta được
\begin{equation} \label{lab9}
\frac{2(27q^2-9q+1)}{9q^2-2q+(1-3q)\sqrt{q(1-3q)}}+\frac{1}{q} - 6 \leqslant (k + 1)^2 + \frac{2}{k+1}.
\end{equation}
Đặt $x = \sqrt{\frac{1-3q}{q}} \geqslant 0$ thì $q = \frac{1}{x^2+3},$ khi đó \eqref{lab9} trở thành
\[\frac{x^3+3x^2+3x+3}{x+1} \leqslant (k + 1)^2 + \frac{2}{k+1},\]
hay
\[(x+1)^2 + \frac{2}{x+1} \leqslant (k + 1)^2 + \frac{2}{k+1},\]
tương đương với
\[\frac{(x-k)\big[(k+1)x^2+(k^2+4k+3)x+k^2+3k\big]}{(x+1)(k+1)} \leqslant 0.\]
Suy ra $x \leqslant k$ hay là
\[(a+b+c)^2\leqslant (k^2+3)(ab+bc+ca),\]
hoặc
\[a^2 + b^2 + c^2 \leqslant (k^2+1)(ab+bc+ca).\]
Bài toán được chứng minh.

 

Nhận xét. Trường hợp $k = 1$ ta được bài toán rất đẹp sau

Nếu $a,\,b,\,c$ là ba số thực dương thỏa mãn
\[\frac{a}{b}+\frac{b}{c}+\frac{c}{a} = 5,\]
thì
\[a^2 + b^2 + c^2 \leqslant 2(ab+bc+ca).\]

 

(Võ Quốc Bá Cẩn)

3. Các bài toán rèn luyện

Để kết thúc chuyên đề xin được giới thiệu một số bài tập để bạn đọc tự luyện

Bài tập 1. Cho $a,\,b,\,c$ là ba số thực dương. Chứng minh rằng
\[\left(\frac{a}{b}+\frac{b}{c}+\frac{c}{a}\right)^3+\frac{525(ab+bc+ca)}{a^2+b^2+c^2} \geqslant \frac{775}{2}.\]

 

(Nguyễn Văn Huyện)

Bài tập 2. Chứng minh rằng với mọi số thực $k \geqslant 12$ ta luôn có
\[\frac{a}{b}+\frac{b}{c}+\frac{c}{a}+\frac{k(ab+bc+ca)}{(a+b+c)^2} \geqslant 4+2\sqrt{k-12},\]
trong đó $a,\,b,\,c$ là ba số thực dương thay đổi bất kỳ.

 

(Tạ Hồng Quảng)

Bài tập 3. Cho $a,\,b,\,c$ là ba số thực dương. Chứng minh rằng
\[\frac{a}{b}+\frac{b}{c}+\dfrac{c}{a} \geqslant \sqrt{\frac{k(a^2+b^2+c^2)}{(a+b+c)^2}+9-\dfrac{k}{3}},\]
trong đó $k = 54\sqrt[3]{2}.$

 

(Bách Ngọc Thành Công)

 

Bài tập 4. Tìm hằng số $k$ lớn nhất sao cho bất đẳng thức
\[\frac{a}{b}+\frac{b}{c}+\frac{c}{a} \geqslant \sqrt{9+k-\frac{k(ab+bc+ca)}{a^2+b^2+c^2}},\]
luôn đúng với mọi số thực dương $a,\,b,\,c.$

Bài tập 5. Tìm hằng số $k$ lớn nhất sao cho bất đẳng thức
\[\frac{a}{b} + \frac{b}{c} + \frac{c}{a} + k \geqslant \sqrt{\frac{k^2(a^2+b^2+c^2)}{ab+bc+ca}} + 3,\]
luôn đúng với mọi số thực dương $a,\,b,\,c$ bất kỳ.

Bài tập 6. Cho $a,\,b,\,c$ là ba số thực dương. Chứng minh rằng
\[\frac{a}{b} + \frac{b}{c} + \frac{c}{a} \geqslant \frac{a^2+b^2+c^2}{ab+bc+ca} + 2 \sqrt{\frac{a^2+b^2+c^2}{ab+bc+ca}}.\]

Bài tập 7. Cho $a,\,b,\,c$ là ba số thực dương. Chứng minh rằng
\[\frac{a}{b} + \frac{b}{c} + \frac{c}{a} + \frac{k(ab+bc+ca)}{a^2+b^2+c^2+ab+bc+ca} \geqslant 3+ \frac{k}{2},\]
trong đó $k = 2\big(3\sqrt[3]{9}-1\big).$

 

(Nguyễn Văn Huyện)

Bài tập 8. Tìm hằng số $k$ lớn nhất sao cho bất đẳng thức
\[\frac{a}{b} + \frac{b}{c} + \frac{c}{a} + k \geqslant \sqrt{\frac{3k^2(a^2+b^2+c^2)}{(a+b+c)^2}} + 3,\]
luôn đúng với mọi số thực dương $a,\,b,\,c$ bất kỳ.

 

(Phạm Sinh Tân)

Bài tập 9. Tìm hằng số $k$ lớn nhất sao cho bất đẳng thức
\[\frac{a}{b} + \frac{b}{c} + \frac{c}{a} + k\left ( \frac{ab+bc+ca}{a^2+b^2+c^2} \right )^2 \geqslant  3+k,\]
luôn đúng với mọi số thực dương $a,\,b,\,c$ bất kỳ.

Bài tập 10. Với $k \geqslant 1$ là một số thực cho trước và $a,\,b,\,c$ là ba số thực dương sao cho
\[\frac{a}{b} + \frac{b}{c} + \frac{c}{a} = \frac{(k^2+9)(a^2+b^2+c^2)}{(a+b+c)^2}.\]
Chứng minh rằng
\[a^2+b^2+c^2 \leqslant (k^2+1)(ab+bc+ca).\]

 

(Nguyễn Văn Huyện)

4. Tài liệu tham khảo

 

  1. Võ Quốc Bá Cẩn, Chuyên Đề Bất Đẳng Thức Hiện Đại, 2008.
  2. Diễn đàn toán học: http://diendantoanhoc.net
  3. Art of Problem Solving: http://artofproblemsolving.com

Attached Files




#615005 Bổ đề hoán vị

Posted by Nguyenhuyen_AG on 14-02-2016 - 18:42 in Tài liệu, chuyên đề, phương pháp về Bất đẳng thức

 bài đọc này bổ ích ghê hình như bài của VMEO4 mở rộng của bổ đề đúng ko ạ ?? thank anh jup em có k thức mới :v

em có bài này nè cũng hay

cho a,b,c >0 chứng minh $a^{3}+b^{3}+c^{3}+3abc\frac{a^{2}b+b^{2}c+c^{2}a}{ab^{2}+bc^{2}+ca^{2}}\geq ab(a+b)+bc(b+c)+ca(c+a)$

 

Bài này anh thấy nó đâu có liên quan gì với bổ đề đâu, em có thể đăng nó trong topic tiếp sức bất đẳng thức sẽ có người giải. :)

 

Bổ đề cồng kềnh quá. Nhớ mà dùng cũng là một vấn đề lớn :D

 

Các bài toán trong này không dùng cho thi Olympic em à. Thật sự thì chúng quá khó đặc biệt là các bài 5, 6, 7, 8, 9 các hằng số $k$ ở đó đều là các hằng số tốt nhất. Bài này anh nghĩ các bạn thích nghiên cứu về bất đẳng thức đọc sẽ thích, còn các bạn muốn thi HSG thì đọc chơi cho biết thôi, đừng bận tâm nhiều quá vì đi thi chả ai cho các dạng bài này. :)




#612624 Bất đẳng thức - Cực trị

Posted by Nguyenhuyen_AG on 03-02-2016 - 10:10 in Bất đẳng thức và cực trị

Bài 55: Cho $x,y,z\geqslant 0$.Chứng minh rằng
$(x^2+y^2+z^2)^2\geqslant 4(x+y+z)(x-y)(y-z)(z-x)$

 

Giả sử $z = \min\{x,\,y,\,z\}.$ Khi đó nếu $x \geqslant y \geqslant z \geqslant 0$ thì

\[(x^2+y^2+z^2)^2 \geqslant 0 \geqslant 4(x+y+z)(x-y)(y-z)(z-x).\]

Còn nếu $y \geqslant x \geqslant z \geqslant 0$ ta đặt

\[f(x,\,y,\,z) = (x^2+y^2+z^2)^2 - 4(x+y+z)(x-y)(y-z)(z-x),\]

và thấy

\[f(x,\,y,\,z) - f(x,\,y,\,0) = z^3+2(x^2+y^2)z+4(y-x)(x^2+xy+y^2-z^2) \geqslant 0.\]

Do đó $f(x,\,y,\,z) \geqslant  f(x,\,y,\,0).$ Công việc còn lại là chứng minh $f(x,\,y,\,0) \geqslant 0,$ nhưng điều này là hiển nhiên vì

\[f(x,\,y,\,0) = (x^2+2xy-y^2)^2 \geqslant 0.\]

Bài toán được chứng minh.




#615077 Bất đẳng thức - Cực trị

Posted by Nguyenhuyen_AG on 14-02-2016 - 21:47 in Bất đẳng thức và cực trị

Biện hộ chỗ này,abc đâu có =1 

 

Chỗ đấy chắc em đó gõ nhầm, bất đẳng thức \[(ab)^{\frac{3}{2}}+(bc)^{\frac{3}{2}}+(ca)^{\frac{3}{2}} \geqslant ab+bc+ca,\] hiển nhiên đúng theo bất đẳng thức trung bình lũy thừa hoặc dùng AM-GM theo kiểu \[(ab)^{\frac{3}{2}}+(ab)^{\frac{3}{2}}+1 \geqslant 3\sqrt[3]{(ab)^{\frac{3}{2}}\cdot(ab)^{\frac{3}{2}}\cdot 1} = 3ab.\]




#615051 Bất đẳng thức - Cực trị

Posted by Nguyenhuyen_AG on 14-02-2016 - 20:59 in Bất đẳng thức và cực trị

Bài 76:

CM: $a^{4}+b^{4}+c^{4}+ab^{3}+bc^{3}+ca^{3}\geq 2(a^{3}b+b^{3}c+c^{3}a)$          (a,b,c là các số thực)

 

Ta có

\[a^{4}+b^{4}+c^{4}+ab^{3}+bc^{3}+ca^{3}-2(a^{3}b+b^{3}c+c^{3}a) = \frac{1}{6}\sum (a^2-2b^2+c^2-ab+2bc-ca)^2,\]

hoặc là

\[a^{4}+b^{4}+c^{4}+ab^{3}+bc^{3}+ca^{3}-2(a^{3}b+b^{3}c+c^{3}a) = \frac{1}{2}\sum (a^2-b^2-ab+bc)^2.\]

Từ đó suy ra điều phải chứng minh.

 

Nhận xét. Thật ra thì bài toán này vẫn có thể chứng minh bằng bất đẳng thức Cauchy-Schwarz.




#615093 Bất đẳng thức - Cực trị

Posted by Nguyenhuyen_AG on 14-02-2016 - 22:19 in Bất đẳng thức và cực trị

Bài 78: $x,y,z\in \mathbb{R}; x+y+z=6$

Tìm max: $P=xy+2yz+3zx$

 

Tổng quát. Với $x,\,y,\,z$ là các số thực thì

\[4(k \cdot xy + t \cdot yz + zx) \leqslant \left [ 1+\frac{(k+t-1)^2}{4t-(t+1-k)^2} \right ](x+y+z)^2,\]

trong đó $k,\,t$ là hai số thực dương cho trước thỏa mãn $4t > (t+1-k)^2.$




#615850 Bất đẳng thức - Cực trị

Posted by Nguyenhuyen_AG on 19-02-2016 - 13:07 in Bất đẳng thức và cực trị

Bài 81: Cho $xy+yz+zx=1$. Tìm min: $x^{2}+2y^{2}+3z^{2}$

 

 Và tìm công thức min tổng quát cho bài trên!

 

Xem ở đây: http://diendantoanho...ào/#entry477645




#618342 Bất đẳng thức - Cực trị

Posted by Nguyenhuyen_AG on 04-03-2016 - 17:07 in Bất đẳng thức và cực trị

Cho các số dương a, b, c thỏa mãn abc=1. Chứng minh rằng: $\frac{a}{\left ( a+1 \right )^{2}}+\frac{b}{\left ( b+1 \right )^{2}}+\frac{c}{\left ( c+1 \right )^{2}}-\frac{4}{(a+1)(b+1)(c+1)}\leq \frac{1}{4}$

 

Thay $(a,\,b,\,c)$ bởi $\left(\frac{a}{b},\,\frac{b}{c},\frac{c}{a}\right)$ thì bất đẳng thức trên trở thành

\[\frac{ab}{(a+b)^2}+\frac{bc}{(b+c)^2}+\frac{ca}{(c+a)^2} \leqslant \frac{1}{4}+\frac{4abc}{(a+b)(b+c)(c+a)}.\]

Thật ra thì ta có đẳng thức sau

\[\frac{ab}{(a+b)^2}+\frac{bc}{(b+c)^2}+\frac{ca}{(c+a)^2} + \frac{(a-b)^2(b-c)^2(c-a)^2}{4(a+b)^2(b+c)^2(c+a)^2} = \frac{1}{4}+\frac{4abc}{(a+b)(b+c)(c+a)}.\] Từ đó suy ra điều phải chứng minh.




#614038 Bất đẳng thức - Cực trị

Posted by Nguyenhuyen_AG on 10-02-2016 - 21:10 in Bất đẳng thức và cực trị

Bài 71:   $x,y,z>0; x+y+z=1$

CM:  $x^{3}+y^{3}+z^{3}+6xyz\geq \frac{1}{3}$         (khá khó)

 

Bất đẳng thức này sai, nhưng nếu sửa lại như vầy thì đúng \[x^{3}+y^{3}+z^{3}+6xyz \geqslant  \frac{1}{4}.\]